Exercice d'un livre sans correction — Les-mathematiques.net The most powerful custom community solution in the world

Exercice d'un livre sans correction

Bonjour
Pourriez-vous me dire si ceci est correct, svp ?

Exo :
Soient deux groupes cycliques $C_m =\, <x>$ et $C_n =\; <y>$ ($m \ne n$)
a) Soit $a \in C_m$ et $b \in C_n$ tels que $o(a) = r$ et $o(b) = s$
Quel est l'ordre de l'élément $(a,b)$ dans $C_m\times C_n$ ?

Ce que j'ai fait :
$o(a) = r \Rightarrow a^r = e$
$o(b) = s \Rightarrow b^s = e$
$o(a,b)$ est la plus petite puissance $q$ pour laquelle $(a,b)^q = (e,e)$
$(e,e) = (a^r, b^s) = ((a^r)^s, (b^s)^r) = (a^{rs}, b^{rs}) = (a,b)^{rs}$

Cependant $rs$ n'est pas la plus petite puissance possible si $r$ et $s$ ont des facteurs communs.
Pour cela il faut que $q = \dfrac{r.s}{r \wedge s}$

Je ne suis pas certains de tout ça car je trouve suspect de ne pas utiliser les cardiaux des groupes $C_m$ et $C_n$.

[En $\LaTeX$, c'est toute l'expression mathématique que l'on encadre par des $\$$, pas seulement quelques termes. AD]

Réponses

  • Le résultat que tu conjecture est exact. Tu peux déjà remarquer que tout multiple de $r$ et de $s$ donne le neutre. Il te reste à identifier la quantité $rs/(r \wedge s)$ d'une autre manière ;-)
  • D'accord, je n'avais pas vu, puis en remplaçant par des valeurs numériques il m'a semblé reconnaître le PPCM, ce que j'ai pu confirmer en surfant et cherchant une relation entre PPCM et PGCD

    Donc l'élément $(a,b)$ est d'ordre PPCM(r,s) dans $C_m \times C_n$

    Merci.
  • Bonjour
    Variante :
    d'abord il n'est pas nécessaire de considérer des groupes cycliques
    si $a,b$ sont respectivement dans les groupes $H,K$
    alors l'ordre de $(a, b)$ est effectivement le plus petit entier $q$ non nul tel que $(a,b)^q=(e_H,e_K)$
    mais l' égalité $(a,b)^q=(e_H,e_K)$ équivaut à $q$ multiple de $r$ et de $s$
    (puisque par exemple $a^q=e_H$ équivaut à $q$ multiple de $r$)

    Donc l'ordre cherché est le plus petit multiple (non nul) de $r$ et $s$
  • C'est vrai que c'est plus clair.

    b) Question suivante :
    A l'aide du résultat précédent, démontrez le corollaire suivant : "Le produit direct de deux groupes cyclique $C_m \times C_n$ est un groupe cyclique si et seulement si m et n sont premiers entre eux."

    Ce que j'ai fait :
    - $C_m = <x>$ est de cardinal m
    - $C_n = <y>$ est de cardinal n
    - $C_{mn} = <z>$ est de cardinal mn
    - Soit $C_m \times C_n$ le produit direct des groupes $C_m$ et $C_n$
    - Soit $z=(x,y)$ un élément de $C_m \times C_n$
    - D'après a), l'élément $z$ est d'ordre $q=PPCM(m,n)$ dans $C_m \times C_n$

    - $m \wedge n \ne 1$ (non premiers entre eux) $ \Rightarrow PPCM(m,n) < mn \Rightarrow C_m \times C_n \not\simeq C_{mn}$ (Car aucun élément $z$ de $C_m \times C_n$ n'est d'ordre $mn$.)
    - $m \wedge n = 1$ (premiers entre eux) $ \Rightarrow PPCM(m,n) = mn \Rightarrow C_m \times C_n \simeq C_{mn}$

    Là où je ne vois pas, c'est que bien que deux groupes soient isomorphes rien me dit que que $C_m \times C_n $ soit cyclique, si ?
    Et même avant ça, je ne suis pas sûr que je peux affirmer (même si je sais que c'est vrai) que bien qu'étant de mêmes cardinaux, $C_m \times C_n \simeq C_{mn}$
  • Si $C_m \times C_n$ est isomorphe à un groupe cyclique (en l'occurrence ça ne peut être que $C_{mn}$ puisqu'il est toujours d'ordre $mn$) alors il est lui-même cyclique. Pour ton autre question, il suffit d'utiliser le fait que si $G$ est un groupe d'ordre $n$ et s'il admet un élément d'ordre $n$ alors $G$ est cyclique, engendré par cet élément.

    Pour résumer : $C_m \times C_n$ est cyclique si et seulement s'il est isomorphe à un groupe cyclique. Si jamais c'est le cas, le groupe cyclique en question ne peut être que $C_{mn}$ pour des raisons de cardinal. Et une manière d'obtenir que $C_m \times C_n$ est cyclique dans le cas où $m \wedge n =1$ est de montrer l'existence d'un élément d'ordre $mn$ dans ce groupe.
  • Poirot a écrit:
    Si $C_m \times C_n$ est isomorphe à un groupe cyclique (en l'occurrence ça ne peut être que Cmn puisqu'il est toujours d'ordre mn) alors il est lui-même cyclique.

    Tout à fait, mais pour l'instant je sais seulement que $C_{mn}$ est cyclique (par définition) mais je n'ai pas prouvé que $C_m \times C_n$ lui était isomorphe, j'ai juste prouvé qu'ils étaient de même cardinalité. C'est ça qui me pose problème.

    Ah , oui, oui, d'accord, je viens de piger, tant pis je laisse mon commentaire ci-dessus. Je vois, merci :)
  • Bonjour Morgatte,

    pour montrer qu'un groupe d'ordre $x$ est isomorphe à $C_x$, il suffit de trouver un élément d'ordre $x$ dans ce groupe.
    Grâce à ta première question, dans le cas où $m$ et $n$ sont premiers entre eux, est-ce que tu arrives un élément d'ordre $mn$ dans $C_m \times C_n$ ?
Connectez-vous ou Inscrivez-vous pour répondre.
Success message!